*ANSWER ASAP* What is the radius of a circle if its center is (-3,-2) and one point on the circle is (5,-2)?

*ANSWER ASAP* What Is The Radius Of A Circle If Its Center Is (-3,-2) And One Point On The Circle Is

Answers

Answer 1

Answer:

8

Step-by-step explanation:

h=-3 , k=-2, x=5, y=-2

(x-h)^2+(y-k)^2 = r^2

(5-(-3))^2+(-2-(-2))^2

64+0=r^2

r=√64=8


Related Questions

can anyone rewrite the quadratic function in standard form?

Answers

Answer:

y = -2(x-2) (x+8)

Step-by-step explanation:

got you bro

Answer:

y = -2x² -12x + 32.

Step-by-step explanation:

Recall that a quadratic equation in standard form is:

y = ax² + bx + c.

To rewrite the equation in standard form, you will need to FOIL and distribute the -2. This can be done by:

y = -2(x-2)(x+8)

FOIL the factored part of the equation. Ignore the '-2' for now.

y = -2 (x² +6x -16)

Now, multiply the '-2' with all of the terms:

y = -2x² -12x + 32.

This is your equation in standard form!

Maria owns 80 shares of a certain stock. Yesterday the value of each share went up by 10 dollars. What was the total change in value of her shares?

Answers

Answer:

800

Step-by-step explanation:

each share is worth 10 more dollars 80x10=800

The letters of the word S U M M E R were placed in a bag. What is the probability of choosing an R?
3:6
5/6
33.3%
0.16

Answers

。☆✼★ ━━━━━━━━━━━━━━  ☾  

There is only one R in the bag

So the probability would be 1/6

1/6 as a decimal is 0.16

Thus, your answer would be option D

Have A Nice Day ❤    

Stay Brainly! ヅ    

- Ally ✧    

。☆✼★ ━━━━━━━━━━━━━━  ☾

I NEED QUICKKKK will mark brainliest A perfectly vertical stack of dominoes has a volume of 1.8 cubic inches. Another stack of the same number of dominoes is slanted slightly to the right. What is its volume?

Answers

Answer: 1.8

Step-by-step explanation: I’m not entirely sure about this, but it probably is 1.8 because I don’t think the slant would change the volume, though I may be wrong

Answer:

1.8 cubic inches

Step-by-step explanation:

The perfectly vertical stack has a volume of 1.8 cubic inches. Therefore, another stack with the same amount would have the same volume, regardless of whether or not it is slanted.

Which of the following indicates the subtraction property of equality when solving the equation 4(x + 9) + 5 = 3x − 8? answers: 4(x + 9) = 3x − 8 − 5 4x + 36 + 5 = 3x − 8 4(x + 9) − 5 + 8 = 3x x = −49

Answers

Answer:  4(x + 9) = 3x − 8 − 5

Step-by-step explanation:

The subtraction property of equality states that f a = b, then a − c = b − c. Subtracting 5 from both sides of the given equation gives 4(x + 9) = 4x − 8 − 5.

In the proportion 15/25=3/5 the terms that are called extremes are?

Answers

Answer:

15 and 5

Step-by-step explanation:

15:25::3:5

so, extreme are the terms which the underlined

Answer:

15, 5

Step-by-step explanation:

The outer terms in a ratio are called extremes. The outer terms here are 15 and 5 while the inner terms or means are 25 and 3.

Find the area of the shaded region of the figure below. Each small square is 1 cm x 1cm.

Answers

Answer: 23 cm squared

Step-by-step explanation:

Since the area of each square is 1 cm squared.

We can count up the number of squares which will be the total area.

There are 23 squares so the area of the region is 23 cm squared.

Answer: 23 cm²

Step-by-step explanation:

Since the squares are each 1 cm × 1 cm, you can technically count all the shaded squares, but it is good to do it mathematically.

We can separate this shape into 1 parts. The top part sticking up is a square and the bottom, a rectangle. We find the area for both parts and add them together.

Square:

3 cm × 3 cm = 9 cm²

Rectangle:

2 cm × 7 cm = 14 cm²

Total:

9 cm² + 14 cm² = 23 cm²

Calculate the Rate of change

Answers

the rate of change is -5. this can be calculated by taking two points, let’s do (0,30) and (3,15). subtract one coordinate from the other (doesn’t matter which one is which) using the formula (y2-y1)/(x2-x1) = rate of change. so that looks like this: (15-30)/(3-0)= -15/3= -5, therefore the rate of change is -5.

Which graph shows the solution to the system of linear inequalities below?

Answers

Answer: Graph A or answer D is correct

Mark is using two different baking pans to bake bread for a bake sale.
If the dimensions of the first pan are 10 x 7 x 3 inches, and the dimensions of the second pan are 10 x 10 x 2 inches, what is the total volume of both baking pans, in cubic inches?

Answers

Answer:

The answer is 410

Step-by-step explanation:

10 x 7 x 3 + 10 x 10 x 2 = 410

looking at the diagrams, answer the question

Answers

Hey!!!

Answers:

1. {d}

2. {a,d,g,f}

3.{g,e,f}

please see attached picture for full solution

Hope it helps

Good luck on your assignment

Stay safe..

What is the value of log6255?
O 4
4 14
O
O 4

Answers

Answer: 3.79622731403

Step-by-step explanation:

It’s really like 3.79622731. Rounded up, 4 so go with D out of your options.

PLEASE HELP! An equilateral triangle has what type of symmetry? A. line symmetry only B. point symmetry only C. both point and line symmetry D. neither point nor line symmetry E. both point and rotational symmetry

Answers

Answer:

A

Step-by-step explanation:

it has 3 lines of symmetry

Answer:

both i think dont be mad if its not right im stressed right now

Step-by-step explanation:

write 0.0004353 in scientific notation

Answers

Answer:

Step-by-step explanation:

4.35 x 10-4

(Scientific Notation)

Answer:

4.353*10^-4

Step-by-step explanation:

Def. of scientific notation

find points of discontinuity y=x^2+5x+6/x^2-16

Answers

Answer:

  x=4 and x=-4

Step-by-step explanation:

We assume you intend ...

  y = (x^2 +5x +6)/(x^2 -16)

There will generally be points of discontinuity where the denominator is zero. Sometimes a denominator factor will be cancelled by a numerator factor, but the "hole" remains and there is still a discontinuity there.

Here, the denominator factors as the difference of squares:

  x^2 -16 = (x -4)(x +4)

There will be discontinuities where these factors are zero, at x=4 and x=-4.

There are 16 liquid measuring cups in a gallon. If a gallon contains 3,785 milliliters, is
the vessel above more or less than a liquid measuring cup?

Answers

Answer: A liquid measuring cup has less liquid than the cup.

Step-by-step explanation: 3785/16=236.5625

Thus, would be less.

Hope this helps <3

-Carrie

P.S. please mark brainliest!

It is found that liquid measuring cup has less liquid than the cup.

What is the unitary method?

The unitary method is a method for solving a problem by the first value of a single unit and then finding the value by multiplying the single value. Unitary method is a technique by which we find the value of a single unit from the value of multiple and the value of more than one unit from the value of a single unit. It is a method that we use for most of the calculations in math.

We are given that 16 liquid measuring cups in a gallon. If a gallon contains 3,785 milliliters,

Therefore, we need to divide the 3,785 milliliters, with the liquid measuring cups

So, 3785/16=236.5625

Thus, That would be less.

Learn more about the unitary method, please visit the link given below;

https://brainly.com/question/23423168

#SPJ3

Which is a pertect square?
72
81
90
99​

Answers

Answer:

81.

Step-by-step explanation:

9 * 9 = 81

Answer:

81

Step-by-step explanation:

81 is a perfect square since

√81 = 9 (perfect square)

However any of the others aren't perfect squares

Which are not changed after a rotation? Check all that apply
angle measures
orientation
size
shape
position of center of rotation

Answers

Answer:

Angle measures, size, shape, position of center of rotation.

Step-by-step explanation:

Rotation is a rigid motion, meaning the angle measures, size, and shape are preserved. Also, since we are rotating around the center, the center would stay the same. So the answer is angle measures, size, shape, position of center of rotation.

When a shape is rotated, it must be rotated through a center.

Angle measures , the size , the shape , and the position of center of rotation do not change after rotation

Rotation is a rigid transformation.

This means that, rotation does not change the angle measure, the side lengths and the shape of the shape that is being rotated.

Except the center of rotation is changed to perform another rotation, the rotation does  not change the center of rotation.

Hence, all except orientation remain unchanged after a rotation.

Read more about rotation at:

https://brainly.com/question/1571997

brainlist if correct
and thanks
question is the screenshot

Answers

Answer:

(0,1/2)

Step-by-step explanation:

When X=0, Y=0.5, or 1/2

1/2=0+1/2

Answer:

(0, 1/2)

Step-by-step explanation:

y = x + 1/2

This is in slope intercept form

y = mx+b where m is the slope and b is the y intercept

The slope is 1 and the y intercept is 1/2

(0, 1/2)

Please help asap, shehhrhd

Answers

Answer:

55°

Step-by-step explanation:

y+x=180°

25°+30°+x=180°

25°+30°=y

y=55°

*Exterior Angle Theorem*

Sorry for the lack of explanation. It's midnight, so I didn't have much time.

Find the common difference, d, in the following arithmetic sequence.
42, 38, 34, 30, 26, …



–5


5


4


–4

Answers

Answer: -4

Step-by-step explanation: Let me know if you need an explanation.

What type of graph is best to use to organize numerical data based on their digits to compare value? A. Venn diagram B. box and whisker plot C. stem-and-leaf plot D. pictograph

Answers

Answer:

B

Step-by-step explanation:

Alex showed me that it was correct

Answer:

B

Step-by-step explanation:

numericaly organize data based on their digits to compare value

Last week, a candle store received $355.60 for selling 20 candles. Small candles sell for $10.98 and large candles sell for $27.98. How many large candles did the store sell?

Answers

Answer:

8 large candles

Step-by-step explanation:

If all the candles were large candles,

Amount received

= 20( $27.98)

= $559.60

Difference in amount received

= $559.60 -$355.60

= $204

Difference in the cost of a small candle to a large candle

= $27.98 -$10.98

= $17

Number of small candles

= $204 ÷$17

= 12

Thus, number of large candles

= 20 -12

= 8

Let's check!

Amount received

= 8( $27.98) +12( $10.98)

= $355.60 ✓

Given that sin 60 = root3/2
Find the exact value of sin 240

Answers

Answer:

[tex]sin 240^\circ = - \dfrac{\sqrt{3}}{2}[/tex]

Step-by-step explanation:

We are given that:

[tex]sin 60^\circ = \dfrac{\sqrt{3}}{2}[/tex]

We need to find

[tex]sin 240^\circ = ?[/tex]

240 is greater than 180 and lesser than 270

OR

[tex]180^\circ<240^\circ<270^\circ[/tex]

So, Angle [tex]240 ^\circ[/tex] lies in the 3rd quadrant and it is well known that value of sine in 3rd quadrant is negative.

Using the property :

[tex]sin(\pi + \theta) = -sin\theta[/tex] or

[tex]sin(180^\circ + \theta) = -sin\theta[/tex]

Here, [tex]\theta\ is\ 60^\circ[/tex].

[tex]sin 240^\circ =sin(180^\circ + 60^\circ) = -sin60^\circ\\\Rightarrow -\dfrac{\sqrt{3}}{2}[/tex]

Hence, the value

[tex]sin 240^\circ = - \dfrac{\sqrt{3}}{2}[/tex]

Express the equation in the form Y= mx+b by solving for Y. -2x-y=10
A: y=-2x+10
B: y=2x+10
C: y= 2x-10
D: y= -2x-10

Answers

Answer:

the answer is d: y=-2x-10

Coverage for injury and property damage sustained by others for which you or covered residents of your household are legally responsible is known as a(n) _____.
a) asset
b) claim
c) debt
d) liability

Answers

b) Claim is the answer to the question
i would say D because it would be a personal liability

What is the domain of the function y=/x?
-00 0 0 1

Answers

Answer:

It Should be (-infinity, + infinity)

Step-by-step explanation:

A mailman earned $832 a week last year. What was his yearly salary?

A. $49,396
B. $50,328
C. $ 47,532
D. $43,264

Answers

D. $43.264 is the correct answer

Answer:

the answer would be d. if i am correct

I need help and an answer ASAP Please don't abandon the answer I need an answer!

Answers

Divide the number of containers by five to get the number of days.

25/5=5, so it takes five days to eat 25 containers.
answer: it would take 5 days to eat 25 yogurts

explanation: if he eats 10 yogurts in 2 days you divide 2 by 2 and 10 by 2 to find how many he eats each day. then you multiply 5 by 5 to get 25 yogurts and 1 by 5 to get five days. i hope this helps!!

QUESTION 5 The temperature was -7°C in the morning. The temperature dropped 3°C by sunset. What was the temperature at sunset? +10°C +4°C -4°C -10°C

Answers

-10°C or you can put positive if you want
Other Questions
Alanco, Inc. manufactures a variety of products and is currently manufacturing all of their own component parts. An outside supplier has offered to sell one of those components to Alanco. To evaluate this offer, the following information has been gathered relating to the cost of producing the component internally:Direct Materials $4Direct Labor $6Variable Manufacturing Overhead $2Fix Manufacturing Overhead, Traceable* $5Fix Manufacturing Overhead, Common but Allocated $8Total Cost $25.00Supplier Price = $21Units Per Year = 12,000Fix manufacturing overhead, traceable is composed of two items:Depreciation of Equipment: 30%Supervisor Salary: 70%Assuming the company has no alternative use for the facilities now being used to produce the component, complete the following analysis to determine if the outside supplier's offer should be accepted.Based on this analysis, write an if statement to determine if Alanco should make or buy the component.Alanco should ............. the component.3 Per Unit Differential Cost 12,000 units Make Buy Make Buy Cost of purchasing Direct materials Direct labor Variable manufacturing overhead Fixed manufacturing overheadtotal costs, Based on this analysis, wrie an if statement to determine if Alanco should make or buy the component. Alanco should the _____________component will mark bianleast Which is the correct ordered pair for point F on the graph?(5, 5)(0, 5)(5, 0)(5, 5) Texture can be used in concert with pattern which is a design based on repetition of grouping of elements such as On October 31, 2021, Damon Companys general ledger shows a checking account balance of $8,415. The companys cash receipts for the month total $74,440, of which $71,325 has been deposited in the bank. In addition, the company has written checks for $72,485, of which $71,090 has been processed by the bank. The bank statement reveals an ending balance of $12,165 and includes the following items not yet recorded by Damon: bank service fees of $210, note receivable collected by the bank of $5,600, and interest earned on the account balance plus from the note of $620. After closer inspection, Damon realizes that the bank incorrectly charged the companys account $540 for an automatic withdrawal that should have been charged to another customers account. The bank agrees to the error. Required: 1. Prepare a bank reconciliation to calculate the correct ending balance of cash on October 31, 2021. (Amounts to be deducted should be indicated with a minus sign.) As evidence supporting the Big Bang theory, what does the redshift of light from galaxies indicate? (1 point)The universe is 13.8 billion years old.The universe is mainly hydrogen.The universe is cooling off.The universe is expanding. Q.2. A cricketer scores the following runs in eight innings :58, 76,40, 35, 46, 45,0, 100What will be their mean score ?a. 400b. 50c. 200Q.3. What will be the range of following data ?32 , 41 , 28, 54, 35, 26,33 , 23, 38, 40b. 23c. 31a. 25 The papers and placards say, that I am to deliver a 4th [of]July oration. This certainly sounds large, and out of thecommon way, for it is true that I have often had theprivilege to speak in this beautiful Hall, and to addressmany who now honor me with their presenceHow does Douglass establish ethos in this passage?A. Douglass points out how little he knows about his audience andthe meaning of Independence DayB. Douglass condescends to his audience about having to speak tothem on a holidayC. Douglass shows his humility at speaking to the audience on thisoccasionD. Douglass expresses his social position in relation to his audience In at least 100 words, discuss whether or not Chang succeeds in making the reader understand his feelings about leaving China. Explain why you feel he does or does not succeed. 1. 10 mi/h = ft/min convert rate The Battles of Okinawa and Iwo Jima were for the U.S.A., butthey showed how conquering Japan would be. Hello I need help on this question. :) solve simultaneous equations 5x+2y=13 2x+6y=26 On a field assignment with her students, Professor Miller observes two male elephant seals engaged in a fierce battle. She explains to her students that they are fighting for mating control of the female harem, a process termed ____. What is the value of x? 7What is ironic about Dee's interest in the churn top and quilts in "Everyday Use" by Alice Walker?She doesn't understand how to use them or where they came from.She wasn't interested in them when they were part of her everyday life.She thought they had been destroyed when her old house burned down.She thinks they are worthless, wom-out, and old-fashioned. When an object rolls over a surface, the kind of friction that occurs is calledfriction.Answer here Ammonia reacts with sulfuric acid to produce the important fertilizer, ammonium hydrogen sulfate.What mass of ammonium hydrogen sulfate can be produced from 75 L of ammonia at 10C and 110Pa?4 NH + H69)thabulan)341 1) HSO4(aq) write a short note on the organs of the government based on thegiven picture ss class 8 1. Jin fills up a 510-gallon pool in the backyard for her children. She fills it with the garden hose at a rate of 17 gallons per minute. After it is filled, she lets it sit for 30 minutes in order to let the water temperature rise. The children then get in and have fun for an hour. The pool loses about _1_ gallon of water each minute due to Time (minutes) Amount of Water (gallons) 0 5 20 30 45 60 80 100 120 150 200 2 their splashing and playing. At the end of the hour, they tear the pool while getting out, which causes a leak. The pool then begins to lose water at a rate of 2 gallons per minute. a. Complete the table to show the amount of water in the pool after each minute. b. Create a graph to model the problem situation. Include when the pool will be empty. c. Write a piecewise function that models this problem situation. Explain your reasoning for each piece of the function. d. Identify the x- and y-intercept. Explain what they mean in terms of the problem situation. e. Determine when the pool will have 470 gallons of water in it. Identify the piece(s) of function you used. Explain your reasoning. 2. Jin asks her children to pay her back for the damaged pool. They must give her $15 per week. Together they have $165 in a savings account. a. Write a function to represent the amount of money they have after x weeks. Describe the domain and range of this function in terms of the problem situation. b. To rebuild their account, the children will receive a combined $15 per week in allowances. They start saving the week after their account is depleted and save for another 11 weeks. What are the domain and range of this function and what do they mean in terms of the problem situation? Write a function to represent this part of the graph. c. Graph the equations on the same grid. Is the graph continuous or discrete? Explain your reasoning. d. Write a function to represent the entire graph. Use the DISTRIBUTIVE PROPERTY to simplify the expression. 4(3r - 8)